If exactly two of the stations stay open, which one of the following must close?

on January 4, 2021

How do you arrive at this answer?

I am confused as to how you make deductions to find this answer!

Reply
Create a free account to read and take part in forum discussions.

Already have an account? log in

Shunhe on January 7, 2021

Hi @bb64,

Thanks for the question! So we assume exactly two stations stay open, which means four total have to close. So we know L and R can’t both be open, and N and R can’t both close. And M is related too. Thinking about their relations, these four could all be in or out. But that means P and Q can’t be in, because two of that set of four have to be in. So P and Q have to be out, and that makes (D) the answer.

Hope this helps! Feel free to ask any other questions that you might have.